Which one of the following, if true, would lend the most credence to the author's statement in lines 56-58?

meisen on May 20, 2018

Question

Why is A wrong and C right?

Replies
Create a free account to read and take part in forum discussions.

Already have an account? log in

meisen on May 20, 2018

Pardon me, why is C wrong and A right?

Mehran on May 20, 2018

Hi @meisen, thanks for your posts.

The question asks you to select an answer choice that would bolster the author's claim at lines 56-58 of the passage ("Here, the provincial court's ruling was excessively conservative in its assessment of the current law."). Answer choice (A) bolsters this claim by establishing that other provincial courts in Ontario had previously adopted a more expansive (i.e., less conservative) view of the current law (with respect to what ownership rights entailed).

Answer choice (C) is incorrect because, if it was true that previous Ontario court decisions had differentiated between the right to use and the right to sell and, then this particular provincial court's decision that the property rights in question extended only to use rights (not also sale rights) would not actually be "excessively conservative." In other words, answer choice (C) would undermine rather than bolster the argument at lines 56-58 of the passage.

Hope this helps! Please let us know if you have any additional questions.